Stay ahead of learning milestones! Enroll in a class over the summer!

G
Topic
First Poster
Last Poster
k a April Highlights and 2025 AoPS Online Class Information
jlacosta   0
Apr 2, 2025
Spring is in full swing and summer is right around the corner, what are your plans? At AoPS Online our schedule has new classes starting now through July, so be sure to keep your skills sharp and be prepared for the Fall school year! Check out the schedule of upcoming classes below.

WOOT early bird pricing is in effect, don’t miss out! If you took MathWOOT Level 2 last year, no worries, it is all new problems this year! Our Worldwide Online Olympiad Training program is for high school level competitors. AoPS designed these courses to help our top students get the deep focus they need to succeed in their specific competition goals. Check out the details at this link for all our WOOT programs in math, computer science, chemistry, and physics.

Looking for summer camps in math and language arts? Be sure to check out the video-based summer camps offered at the Virtual Campus that are 2- to 4-weeks in duration. There are middle and high school competition math camps as well as Math Beasts camps that review key topics coupled with fun explorations covering areas such as graph theory (Math Beasts Camp 6), cryptography (Math Beasts Camp 7-8), and topology (Math Beasts Camp 8-9)!

Be sure to mark your calendars for the following events:
[list][*]April 3rd (Webinar), 4pm PT/7:00pm ET, Learning with AoPS: Perspectives from a Parent, Math Camp Instructor, and University Professor
[*]April 8th (Math Jam), 4:30pm PT/7:30pm ET, 2025 MATHCOUNTS State Discussion
April 9th (Webinar), 4:00pm PT/7:00pm ET, Learn about Video-based Summer Camps at the Virtual Campus
[*]April 10th (Math Jam), 4:30pm PT/7:30pm ET, 2025 MathILy and MathILy-Er Math Jam: Multibackwards Numbers
[*]April 22nd (Webinar), 4:00pm PT/7:00pm ET, Competitive Programming at AoPS (USACO).[/list]
Our full course list for upcoming classes is below:
All classes run 7:30pm-8:45pm ET/4:30pm - 5:45pm PT unless otherwise noted.

Introductory: Grades 5-10

Prealgebra 1 Self-Paced

Prealgebra 1
Sunday, Apr 13 - Aug 10
Tuesday, May 13 - Aug 26
Thursday, May 29 - Sep 11
Sunday, Jun 15 - Oct 12
Monday, Jun 30 - Oct 20
Wednesday, Jul 16 - Oct 29

Prealgebra 2 Self-Paced

Prealgebra 2
Sunday, Apr 13 - Aug 10
Wednesday, May 7 - Aug 20
Monday, Jun 2 - Sep 22
Sunday, Jun 29 - Oct 26
Friday, Jul 25 - Nov 21

Introduction to Algebra A Self-Paced

Introduction to Algebra A
Monday, Apr 7 - Jul 28
Sunday, May 11 - Sep 14 (1:00 - 2:30 pm ET/10:00 - 11:30 am PT)
Wednesday, May 14 - Aug 27
Friday, May 30 - Sep 26
Monday, Jun 2 - Sep 22
Sunday, Jun 15 - Oct 12
Thursday, Jun 26 - Oct 9
Tuesday, Jul 15 - Oct 28

Introduction to Counting & Probability Self-Paced

Introduction to Counting & Probability
Wednesday, Apr 16 - Jul 2
Thursday, May 15 - Jul 31
Sunday, Jun 1 - Aug 24
Thursday, Jun 12 - Aug 28
Wednesday, Jul 9 - Sep 24
Sunday, Jul 27 - Oct 19

Introduction to Number Theory
Thursday, Apr 17 - Jul 3
Friday, May 9 - Aug 1
Wednesday, May 21 - Aug 6
Monday, Jun 9 - Aug 25
Sunday, Jun 15 - Sep 14
Tuesday, Jul 15 - Sep 30

Introduction to Algebra B Self-Paced

Introduction to Algebra B
Wednesday, Apr 16 - Jul 30
Tuesday, May 6 - Aug 19
Wednesday, Jun 4 - Sep 17
Sunday, Jun 22 - Oct 19
Friday, Jul 18 - Nov 14

Introduction to Geometry
Wednesday, Apr 23 - Oct 1
Sunday, May 11 - Nov 9
Tuesday, May 20 - Oct 28
Monday, Jun 16 - Dec 8
Friday, Jun 20 - Jan 9
Sunday, Jun 29 - Jan 11
Monday, Jul 14 - Jan 19

Intermediate: Grades 8-12

Intermediate Algebra
Monday, Apr 21 - Oct 13
Sunday, Jun 1 - Nov 23
Tuesday, Jun 10 - Nov 18
Wednesday, Jun 25 - Dec 10
Sunday, Jul 13 - Jan 18
Thursday, Jul 24 - Jan 22

Intermediate Counting & Probability
Wednesday, May 21 - Sep 17
Sunday, Jun 22 - Nov 2

Intermediate Number Theory
Friday, Apr 11 - Jun 27
Sunday, Jun 1 - Aug 24
Wednesday, Jun 18 - Sep 3

Precalculus
Wednesday, Apr 9 - Sep 3
Friday, May 16 - Oct 24
Sunday, Jun 1 - Nov 9
Monday, Jun 30 - Dec 8

Advanced: Grades 9-12

Olympiad Geometry
Tuesday, Jun 10 - Aug 26

Calculus
Tuesday, May 27 - Nov 11
Wednesday, Jun 25 - Dec 17

Group Theory
Thursday, Jun 12 - Sep 11

Contest Preparation: Grades 6-12

MATHCOUNTS/AMC 8 Basics
Wednesday, Apr 16 - Jul 2
Friday, May 23 - Aug 15
Monday, Jun 2 - Aug 18
Thursday, Jun 12 - Aug 28
Sunday, Jun 22 - Sep 21
Tues & Thurs, Jul 8 - Aug 14 (meets twice a week!)

MATHCOUNTS/AMC 8 Advanced
Friday, Apr 11 - Jun 27
Sunday, May 11 - Aug 10
Tuesday, May 27 - Aug 12
Wednesday, Jun 11 - Aug 27
Sunday, Jun 22 - Sep 21
Tues & Thurs, Jul 8 - Aug 14 (meets twice a week!)

AMC 10 Problem Series
Friday, May 9 - Aug 1
Sunday, Jun 1 - Aug 24
Thursday, Jun 12 - Aug 28
Tuesday, Jun 17 - Sep 2
Sunday, Jun 22 - Sep 21 (1:00 - 2:30 pm ET/10:00 - 11:30 am PT)
Monday, Jun 23 - Sep 15
Tues & Thurs, Jul 8 - Aug 14 (meets twice a week!)

AMC 10 Final Fives
Sunday, May 11 - Jun 8
Tuesday, May 27 - Jun 17
Monday, Jun 30 - Jul 21

AMC 12 Problem Series
Tuesday, May 27 - Aug 12
Thursday, Jun 12 - Aug 28
Sunday, Jun 22 - Sep 21
Wednesday, Aug 6 - Oct 22

AMC 12 Final Fives
Sunday, May 18 - Jun 15

F=ma Problem Series
Wednesday, Jun 11 - Aug 27

WOOT Programs
Visit the pages linked for full schedule details for each of these programs!


MathWOOT Level 1
MathWOOT Level 2
ChemWOOT
CodeWOOT
PhysicsWOOT

Programming

Introduction to Programming with Python
Thursday, May 22 - Aug 7
Sunday, Jun 15 - Sep 14 (1:00 - 2:30 pm ET/10:00 - 11:30 am PT)
Tuesday, Jun 17 - Sep 2
Monday, Jun 30 - Sep 22

Intermediate Programming with Python
Sunday, Jun 1 - Aug 24
Monday, Jun 30 - Sep 22

USACO Bronze Problem Series
Tuesday, May 13 - Jul 29
Sunday, Jun 22 - Sep 1

Physics

Introduction to Physics
Wednesday, May 21 - Aug 6
Sunday, Jun 15 - Sep 14
Monday, Jun 23 - Sep 15

Physics 1: Mechanics
Thursday, May 22 - Oct 30
Monday, Jun 23 - Dec 15

Relativity
Sat & Sun, Apr 26 - Apr 27 (4:00 - 7:00 pm ET/1:00 - 4:00pm PT)
Mon, Tue, Wed & Thurs, Jun 23 - Jun 26 (meets every day of the week!)
0 replies
jlacosta
Apr 2, 2025
0 replies
binomial sum ratio
thewayofthe_dragon   3
N 29 minutes ago by P162008
Source: YT
Someone please evaluate this ratio inside the log for any given n(I feel the sum doesn't have any nice closed form).
3 replies
+1 w
thewayofthe_dragon
Jun 16, 2024
P162008
29 minutes ago
IMO Shortlist 2013, Combinatorics #3
lyukhson   31
N 43 minutes ago by Maximilian113
Source: IMO Shortlist 2013, Combinatorics #3
A crazy physicist discovered a new kind of particle wich he called an imon, after some of them mysteriously appeared in his lab. Some pairs of imons in the lab can be entangled, and each imon can participate in many entanglement relations. The physicist has found a way to perform the following two kinds of operations with these particles, one operation at a time.
(i) If some imon is entangled with an odd number of other imons in the lab, then the physicist can destroy it.
(ii) At any moment, he may double the whole family of imons in the lab by creating a copy $I'$ of each imon $I$. During this procedure, the two copies $I'$ and $J'$ become entangled if and only if the original imons $I$ and $J$ are entangled, and each copy $I'$ becomes entangled with its original imon $I$; no other entanglements occur or disappear at this moment.

Prove that the physicist may apply a sequence of such operations resulting in a family of imons, no two of which are entangled.
31 replies
lyukhson
Jul 9, 2014
Maximilian113
43 minutes ago
A wizard kidnaps 101 people
Leicich   14
N an hour ago by MathCosine
Source: Argentina TST 2011, Problem 2
A wizard kidnaps $31$ members from party $A$, $28$ members from party $B$, $23$ members from party $C$, and $19$ members from party $D$, keeping them isolated in individual rooms in his castle, where he forces them to work.
Every day, after work, the kidnapped people can walk in the park and talk with each other. However, when three members of three different parties start talking with each other, the wizard reconverts them to the fourth party (there are no conversations with $4$ or more people involved).

a) Find out whether it is possible that, after some time, all of the kidnapped people belong to the same party. If the answer is yes, determine to which party they will belong.
b) Find all quartets of positive integers that add up to $101$ that if they were to be considered the number of members from the four parties, it is possible that, after some time, all of the kidnapped people belong to the same party, under the same rules imposed by the wizard.
14 replies
Leicich
Aug 29, 2014
MathCosine
an hour ago
PAMO Problem 4: Perpendicular lines
DylanN   11
N an hour ago by ATM_
Source: 2019 Pan-African Mathematics Olympiad, Problem 4
The tangents to the circumcircle of $\triangle ABC$ at $B$ and $C$ meet at $D$. The circumcircle of $\triangle BCD$ meets sides $AC$ and $AB$ again at $E$ and $F$ respectively. Let $O$ be the circumcentre of $\triangle ABC$. Show that $AO$ is perpendicular to $EF$.
11 replies
DylanN
Apr 9, 2019
ATM_
an hour ago
No more topics!
Why is the old one deleted?
EeEeRUT   14
N Yesterday at 10:08 AM by zRevenant
Source: EGMO 2025 P1
For a positive integer $N$, let $c_1 < c_2 < \cdots < c_m$ be all positive integers smaller than $N$ that are coprime to $N$. Find all $N \geqslant 3$ such that $$\gcd( N, c_i + c_{i+1}) \neq 1$$for all $1 \leqslant i \leqslant m-1$

Here $\gcd(a, b)$ is the largest positive integer that divides both $a$ and $b$. Integers $a$ and $b$ are coprime if $\gcd(a, b) = 1$.

Proposed by Paulius Aleknavičius, Lithuania
14 replies
EeEeRUT
Apr 16, 2025
zRevenant
Yesterday at 10:08 AM
Why is the old one deleted?
G H J
G H BBookmark kLocked kLocked NReply
Source: EGMO 2025 P1
The post below has been deleted. Click to close.
This post has been deleted. Click here to see post.
EeEeRUT
64 posts
#1 • 3 Y
Y by dangerousliri, R8kt, MuhammadAmmar
For a positive integer $N$, let $c_1 < c_2 < \cdots < c_m$ be all positive integers smaller than $N$ that are coprime to $N$. Find all $N \geqslant 3$ such that $$\gcd( N, c_i + c_{i+1}) \neq 1$$for all $1 \leqslant i \leqslant m-1$

Here $\gcd(a, b)$ is the largest positive integer that divides both $a$ and $b$. Integers $a$ and $b$ are coprime if $\gcd(a, b) = 1$.

Proposed by Paulius Aleknavičius, Lithuania
This post has been edited 2 times. Last edited by EeEeRUT, Apr 18, 2025, 12:56 AM
Reason: Authorship
Z K Y
The post below has been deleted. Click to close.
This post has been deleted. Click here to see post.
MathLuis
1501 posts
#2 • 1 Y
Y by R8kt
We claim that all $N$ even and power of $3$ work.
To see they do for $N$ even it is trivial as all $c_i$'s are odd and for $N$ power of $3$ just notice the $c_i$'s cycle between being $1,2 \pmod 3$ and thus the sum of consecutive terms is always divisible by $3$.
Now suppose $N$ was odd but not a power of $3$ then notice $c_1=1, c_2=2$ so $3 \mid N$ and thus we can consider $N=3^k \cdot \ell$ for $k \ge 1$ and if $\ell=3m+1$ then notice $3m-1, 3m+2$ are both coprime to $\ell$ and are consecutive coprimes for obvious reasons so we must have $\gcd(N, 6m+1)>1$ however if they did share a prime divisor then from euclid alg it divides $3^k$ and thus it has to be $3$ which is a contradiction, a similar thing can be done for $\ell=3m+2$ thus we are done :cool:.
Z K Y
The post below has been deleted. Click to close.
This post has been deleted. Click here to see post.
ItzsleepyXD
108 posts
#3 • 1 Y
Y by R8kt
Ans is all $N$ even and $N =3^m$ .
note that all even is true (easy to see)
claim 1 if $2 \mid N+1$ then $3 \mid N$
after that is easy (I am lazy to retype the solution again)
Z K Y
The post below has been deleted. Click to close.
This post has been deleted. Click here to see post.
NicoN9
123 posts
#4 • 2 Y
Y by R8kt, shafikbara48593762
Why is that deleted? rewriting!

The answer is all even number, and power of $3$ (and $N\ge 3$.) These $N$ works since if $N$ is even, then all $c_i$ are odd, and if $N$ is power of $3$, then $c_1, c_2, c_3, \dots$ are $1, 2, 1, 2, \dots \pmod 3$, which works.

Also, we see that if $N$ odd, then $c_1=1$ and $c_2=2$, thus $3\mid N$. Assume for a contradiction that $N$ has prime factors $3<p_1<\dots <p_r$ and let $P=p_1p_2\dots p_r$.

$\bullet$ if $P\equiv 1\pmod 3$, then there exists an integer $h$ with $c_h=P-2$, and $c_{h+1}=P+1$. Now,\[
\gcd(N, c_h+c_{h+1})=\gcd(N, 2P-1)=1.
\]contradiction.

$\bullet$ similarly for $P\equiv 2\pmod 3$, we have $c_k=P-1$ and $c_{k+1}=P+2$. $\gcd(N, 2P+1)=1$.

So we are done.
This post has been edited 1 time. Last edited by NicoN9, Apr 16, 2025, 8:05 AM
Reason: 3 divides N
Z K Y
The post below has been deleted. Click to close.
This post has been deleted. Click here to see post.
MuradSafarli
86 posts
#5 • 1 Y
Y by R8kt
Let’s consider two cases depending on whether \( N \) is even or odd:

Case 1: \( N \) is even.
In this case, any number coprime with \( N \) will be odd. Therefore, the sum \( C_i + C_{i+1} \) will be even, which means the GCD of \( N \) and \( C_i + C_{i+1} \) will always be greater than 1. So, the condition is satisfied for all even \( N \).

Now, let’s analyze the odd case.

Let \( C_1 = 1 \), \( C_2 = 2 \). According to the problem condition, \( \gcd(N, 3) > 1 \).
This implies \( N \) must be divisible by 3. Trying some small odd values divisible by 3, we see that \( N = 3, 9, 27 \) satisfy the condition, while \( N = 15, 21 \) do not.

So we explore two subcases:
Case 2.1: \( N = 3^a \)

In this form, every number congruent to 1 or 2 mod 3 is coprime with \( N \).
Let \( C_j = 3t + 1 \), then \( C_{j+1} = 3t + 2 \), so the sum \( C_j + C_{j+1} = 3t + 1 + 3t + 2 = 6t + 3 \), which is divisible by 3.
Similarly, for \( C_j = 3t + 2 \), we have \( C_{j+1} = 3t + 4 \), so \( C_j + C_{j+1} = 6t + 6 \), again divisible by 3.
Thus, any \( N = 3^a \), where \( a \) is a positive integer, satisfies the condition.

Case 2.2: \( N = 3^a \cdot k \), where \( \gcd(k, 3) = 1 \).
Then \( k \equiv 1 \) or \( 2 \mod 3 \).

Case 2.2.1: Let \( k \equiv 2 \mod 3 \).
Since \( N \) is odd, \( k \equiv 5 \mod 6 \), so write \( k = 6t + 5 \).
There exists an integer \( h\) such that \( C_h = 6f + 4 \).
Here, \( \gcd(6f + 4, 3) = 1 \), and \( \gcd(6f + 5, 6f + 4) = 1 \).
Then \( C_{h+1} = 6f + 7 \), and clearly \( \gcd(6f + 7, 6f + 5) = 1 \), \( \gcd(6f + 7, 3) = 1 \).
Now the sum \( C_h + C_{h+1} = 12f + 11 \).
Then,
\[
\gcd(12f + 11, N) = \gcd(12f + 11, 3^a \cdot (6f + 5)) = \gcd(12f + 11, 6f + 5)
\]\[
= \gcd(6f + 6, 6f + 5) = 1 \quad \text{(Contradiction!)}
\]Case 2.2.2:\( k \equiv 1 \mod 3 \), i.e., \( k = 6f + 1 \).
Then take \( C_h = 6f - 1 \), and \( C_{h+1} = 6f + 2 \).
Now:
\[
\gcd(N, C_h + C_{h+1}) = \gcd(12f + 1, N) = \gcd(12f + 1, 6f + 1) = \gcd(6f, 6f + 1) = 1 \quad \text{(Contradiction!)}
\]Final Answer:
1) \( N = 2k \), for any natural number \( k > 1 \)
2) \( N = 3^a \), for any natural number \( a \)
Z K Y
The post below has been deleted. Click to close.
This post has been deleted. Click here to see post.
SimplisticFormulas
97 posts
#6 • 1 Y
Y by R8kt
overcomplication be like
Z K Y
The post below has been deleted. Click to close.
This post has been deleted. Click here to see post.
Safal
168 posts
#7 • 1 Y
Y by R8kt
My Solution
This post has been edited 6 times. Last edited by Safal, Apr 17, 2025, 7:08 PM
Z K Y
The post below has been deleted. Click to close.
This post has been deleted. Click here to see post.
de-Kirschbaum
196 posts
#8 • 1 Y
Y by R8kt
We claim that the solutions are $N=2^kt, 3^k$.

First note that if $2 \mid N$ then we only have odd numbers left, and odd plus odd must be even so every $N=2^kt$ works. If $2 \nmid N$ then we know that $1=c_1, 2=c_2$ so $1+2=3 \mid N$. If $N=3^k$ then $c_1, c_2, \ldots , c_m \equiv 1, 2, \ldots \mod{3}$ and clearly $3 \mid c_i+c_{i+1}$ for all $i$ so $N=3^k$ also works.

If $N=3^kt$ where $2,3 \nmid t \neq 1$ then we know that there is some $h \in \{1,2\}$ such that $ht \equiv 2 \mod{3}$. Then, $ht$ won't be in $\{c_1,\ldots,c_m\}$ because it isn't coprime with $t$, $ht+1$ is coprime with $t$ but will be $0 \mod{3}$ so it won't be in $\{c_1,\ldots,c_m\}$ either. $ht+2$ will be $1 \mod {3}$ and $(ht+2,t)=(2,t)=1$ so $ht+2$ is coprime with $N$. Similarly $ht-1$ is coprime with $t$ and is $1 \mod{3}$ so $ht-1$ is coprime with $N$ and $ht-1, ht+2$ are actually consecutive coprime numbers, so we can take $ht-1+ht+2 =2ht+1$. This is clearly coprime with $t$ and it is $2 \mod{3}$ so it is coprime with $N$. Thus no $3^kt$ works if $2, 3 \nmid t$. So the only solutions are $N=2^kt, 3^k$.

(It's impossible for $ht+2 \leq 2t+2$ to be out of range because $2t+2 < 3t \leq N \implies 2<t$, since $t=1,2$ are both impossible this construction is always safe)
This post has been edited 1 time. Last edited by de-Kirschbaum, Apr 17, 2025, 8:09 PM
Z K Y
The post below has been deleted. Click to close.
This post has been deleted. Click here to see post.
MathematicalArceus
35 posts
#9 • 1 Y
Y by R8kt
Answers: The only possible solutions are $N=3^k$ and $N=2k \quad \forall k$.

$N=2k$ Note that obviously this works, because $c_i \in \{2k+1\}$ and sum of any odd number is always even and hence $c_i+c_{i+1}=2m \implies \gcd(2m,2k)>1$, the condition, we required.

$N=3^k$ Note that taking the set $\{c_i\}$ and writing the set under $\pmod{3}$ gives us: $\{1,-1,1,-1,\dots -1\}$ so for any $i$, $c_i+c_{i+1} \equiv 0 \pmod{3}$, granting our condition to be true.

No other sols exist: We only consider for odd $N$, because all even $N$ works. Note that if any odd $N$ works, this means $3 \mid N$ because for any odd $N, \text{ } c_1+c_2=1+2=3 \implies \gcd(N,3)>1 \implies 3 \mid N$. Now, let $N=3^kp_1^{a_1}p_2^{a_2}\dots p_n^{a_n}$. We consider $p_1p_2\dots p_n \pmod{3}$.

$p_1p_2\dots p_n \equiv 1 \pmod{3}$ Note this implies that $p_1p_2\dots p_n -2 \equiv 2 \pmod{3}$ and $p_1p_2\dots p_n+1 \equiv 2 \pmod{3}$. Adding we get $2p_1p_2\dots p_n-1 \equiv 1 \pmod{3}$, which is obviously coprime to $N$. Note that $c_i = p_1p_2\dots p_n-2, c_{i+1} = p_1p_2\dots p_n+1$ and hence we get $c_i+c_{i+1}$, such that its gcd with $N$ is 1.

Similarly $p_1p_2\dots p_n \equiv 2 \pmod{3}$ follow the same argument and hence, we are done!

Remark
This post has been edited 2 times. Last edited by MathematicalArceus, Apr 17, 2025, 7:41 PM
Z K Y
The post below has been deleted. Click to close.
This post has been deleted. Click here to see post.
Jupiterballs
42 posts
#10 • 2 Y
Y by R8kt, GeoKing
Just for the sake of not typing twice, here is a pdf solution :gleam:
Attachments:
EGMO 2025 Solution.pdf (32kb)
Z K Y
The post below has been deleted. Click to close.
This post has been deleted. Click here to see post.
kamatadu
478 posts
#11 • 2 Y
Y by R8kt, SilverBlaze_SY
Solved with SilverBlaze_SY.

We prove that $N=2k$ for $k\ge 2$ and $N=3^k$ for $k\ge 1$ are the only solutions.

First we show that these work.

For $N=2k$, note that all the integers coprime to $N$ are odd and summing two of them would give an even result. So the $\gcd$ is at least $2$ and we are done.

For $N=3^{k}$, note that the integers coprime to it alternate between $1$ and $2\pmod 3$. This means that sum of two consecutive integers among them is going to be divisible by $3$ which makes the $\gcd$ at least $3$ and we are done.

Now we show that the other cases are not possible. FTSOC assume that some other integer which is not even or a power of $3$ works.

Note that clearly $\gcd(N,1)=1$ and $\gcd(N,2)=1$ as $N$ is odd. Since we must have $\gcd(N,3)\not=1$, we get $3\mid N$.

Represent $N=3^{\alpha_1}\cdot q$ where $\alpha_1\ge 1$ and $\gcd(q,3) = 1$, $q\not=1$.

If $q\equiv 1\pmod 3$, then note that $\gcd(N,q-2)=\gcd(N,q+1)=1$. Also, $\gcd(N,(q-2)+(q+1))=\gcd(N, 2q-1) = 1$ which gives a contradiction.

If $q\equiv 2\pmod 3$, then note that $\gcd(N, q-1) = \gcd(N, q+2)=1$. Also, $\gcd(N,(q-1)+(q+2))=\gcd(N, 2q+1)=1$ which gives a contradiction too.

Therefore such a choice of $N$ is not possible and we are done.
This post has been edited 1 time. Last edited by kamatadu, Apr 18, 2025, 2:22 PM
Z K Y
The post below has been deleted. Click to close.
This post has been deleted. Click here to see post.
Mathgloggers
68 posts
#12
Y by
WE can use a type of "infinite ascent" here to show that only $3^k$ and all even numbers is the only solution:
FTSOC,consider
$N=3^K.p_1^{a_1}...p_i{a_i}$,but there must exist some $x>N$ for which :$gcd(3x+1,N) \neq 1\implies gcd(3x-1,N)=1=gcd(3x+2,N)$,hence we must have :
The prime dividing $3x-1+3x+2=6x+1$ in the prime list of our $N$ ,but here we see that $gcd(6x+1,N)=1$ hence we have to include another prime $q$ which is not in our list hence we can make our list infinitely big hence no numbers like this exists.

Why $(6x+1,N) =1$ as explained by Luis above also that if we continue on applying the EDA,we would reach somewhere : $6x+1|3^k.(3x+1)$ which is not not possible.
This post has been edited 2 times. Last edited by Mathgloggers, Apr 19, 2025, 9:32 AM
Reason: n
Z K Y
The post below has been deleted. Click to close.
This post has been deleted. Click here to see post.
John_Mgr
67 posts
#13
Y by
Claim: The only solution are N is even number and $N=3^k$ $\forall$$k\ge1$
Proof:
For N is even:
$c_i\equiv 1,2(mod3)$ as $gcd(N,c_i)=1$
i.e if $c_i\equiv 1(mod3)$ then, $c_{i+1}\equiv 2 (mod3)$ or vice versa $\implies gcd(N,c_i + c_{i+1})>1$
For $N=3^k$, similar to above
For $N=3^k\cdot t$ and $(3,t)=1$ $t>1$
case 1: $t\equiv1(mod3)$
$3\mid t-1$ so $3\nmid (t-2, t+1) $ $\implies gcd(3^k\cdot t, (t-2)+(t+1))=gcd(3^k\cdot t, 2t-1)=1$, we are done..
case 2:$t\equiv 2(mod3)$
$3\mid t-2$ and $3\nmid (t-1, t+2)$ $\implies gcd(3^k\cdot t,(t-1)+(t+2))=gcd(3^k\cdot t, 2t+1)=1$, we are done...
Z K Y
The post below has been deleted. Click to close.
This post has been deleted. Click here to see post.
EVKV
57 posts
#14
Y by
CASE 1 N is even

Claim: All even N works
Proof: As all $c_{i}$ are even the sum of 2 consecutive co-prime 2 divides the gcd thus its not 1

CASE 2 N is odd

Claim: 3|N
Proof: 1,2 are consecutive co primes and so gcd(N,3)$\neq$1 so 3|N

Claim: Only prime dividing N is 3
Proof: Assume the contrary lets assume $q_{1}, \cdots ,q_{n}$ are all the primes except 3 which divide N

Claim if $\prod q_{i}$ $\equiv 1$ mod 3 then there exists an r such that the product divides 3r+1 and $ 3r+1 \leq N$

Proof: clearly 3r+1= $\prod q_{i}$ $\leq N$ satisfies this

Claim if $\prod q_{i}$ $\equiv 2$ mod 3 then there exists an r such that the product divides 3r+1 and $ 3r+1 \leq N$

Proof: clearly 3r+1= $2\prod q_{i}$ $\leq N$ satisfies this

Now 3(r-1)+2 and 3r +2 are consecutive co primes
3(r-1)+2 + 3r +2 $\equiv -2+1$ $\equiv 1$ mod $q_{i}$ for all i $\leq n$
3(r-1)+2 + 3r +2 $\equiv 2+2$ $\equiv 1$ mod $3$
Thus gcd(N,3(r-1)+2 + 3r +2)=1
Contradiction

Claim: N=$3^a$ for $a \in N$

Proof: Clearly the consecutive co primes are $1+2 \equiv 0$ mod 3 thus gcd(N,$c_{i} + c_{i+1}$) $\neq1$



REMARK:
A nice problem tho I changed my solution making it more rigorous crt was def grt motivation

I rate it 5 mohs

SOLVED ON 21/4/25
ALSO 50TH POST YAY
This post has been edited 2 times. Last edited by EVKV, Tuesday at 5:42 PM
Z K Y
The post below has been deleted. Click to close.
This post has been deleted. Click here to see post.
zRevenant
11 posts
#15
Y by
(Livesolved on YouTube: Art of Olympiad Mathematics)

Answer: $n$ is even or a power of $3$.

Proof. If $n$ is even, then all $c_i$ are all odd, meaning that $c_i + c_{i+1}$ is even, which is never coprime to $n$ so it works. If $n=3^k$, then all the $c_i$'s are just numbers that are not divisible by $3$ meaning that the residues go $1, 2, 1, 2, ... $ mod $3$. Hence this works as well.

Now, let's suppose $n=3^k \cdot a$. Then we look at $a+1$ and $a-1$. At least one of them is not divisible by $3$, and by looking at the sums of adjacent ones we are done - either we get $a+(a+2)=2a+2$ which is coprime to $n$ if $3 | a+1$ or we get $a$+$a-2$ which works as well.
Z K Y
N Quick Reply
G
H
=
a